Section Name: Analytical And Numerical Ability

  • Uploaded by: rupasree dey
  • 0
  • 0
  • January 2021
  • PDF

This document was uploaded by user and they confirmed that they have the permission to share it. If you are author or own the copyright of this book, please report to us by using this DMCA report form. Report DMCA


Overview

Download & View Section Name: Analytical And Numerical Ability as PDF for free.

More details

  • Words: 17,764
  • Pages: 67
Loading documents preview...
Section Name : Analytical and Numerical Ability Type: MCQ Single    |    Marks: +1/-0   |    Topic: Quant > Percentages    |    Time Spent:  1 Mins 14 Secs 1.  In a class, 55% of the students play cricket, 60% play volley ball and 70% play foot ball. Each student plays at least one of the three games. What is the maximum possible percentage of students who play all the three games? A. 42 B. 43 C. 85 D. 55 Status :   Incorrect Given Answer : C Answer : A View Solution Let 'a' represent the people who play exactly one game, 'b' represent the people who play exactly two games and 'C' repesents, the people who play exactly three games. Thus, a + b + c = 100 ....(1) and a + 2b + 3c = 185 ....(2) From (1) and (2) b + 2c = 85 to maximize 'C' we have to minimize 'b' the minimum value, for b is 1, as 2C must be an integer. Thus, C = 42 Bookmark Share question feedback Type: MCQ Single    |    Marks: +1/-0   |    Topic: Quant > Geometry    |    Time Spent:  1 Mins 14 Secs 2. 

In the given figure, ABC is a triangle. BCP is a straight line. What is the value of x? A. 30o B. 45o C. 40o D. 35o Status :   Correct

Answer : C View Solution In any triangle exterior angle = sum of the interior opposite angles, x + 2x = 120 x = 40o Bookmark Share question feedback Type: MCQ Single    |    Marks: +1/-0   |    Topic: Quant > Average    |    Time Spent:  0 Mins 48 Secs 3.  The average age of students of a class and their class teacher, who is 44 years old, is 22 years. Which of the following can be the average age (in years) of the students? A. 22.5 B. 23 C. 21 D. 20.5 Status :   Incorrect Given Answer : D Answer : C View Solution Let the number of the students in the class be N. Let the average age of the students be A years. Average age of the to class of the students and their class teacher = \frac

{ AN+44 }{

N+1 } =22N+1AN+44=22 AN+44=22N+22 AN=22 (N-1) N > N-1 Therefore AN=22(N-1) => A > 22 Therefore among the choices, A can be 21 or 20.5 But N must be an integer. A=21 results in N=22 which A=20.5 results in N = \frac possible

{ 44 }{ 3 }344 which is not

Therefore A = 21 Bookmark Share question feedback Type: MCQ Single    |    Marks: +1/-0   |    Topic: Quant > Average    |    Time Spent:  1 Mins 6 Secs 4. 

A get-together was attended by 40 gentlemen and 70 ladies. The average height of the gentlemen was 183 cm. The average height of all the persons was 169 cm. Find the average height (in cm) of the ladies. A. 161 B. 163 C. 159 D. 157 Status :   Correct Answer : A View Solution Let the average height of the ladies be \ellℓ cm. Average height of all the persons = \frac

{ 183.40+\ell 70 }{ 40+70 } cm=169cm40+70183.40+ℓ70cm=169cm

183.40+70l = 169 (110) 7320 =70l = 18590 70l = 11270 l = 161 Bookmark Share question feedback Type: MCQ Single    |    Marks: +1/-0   |    Topic: Quant > Mixtures And Alligations    |   Time Spent:  2 Mins 42 Secs 5.  A vessel has 90 litres of pure milk. 9 litres of its contents was withdrawn and replaced with water. This process was repeated once. Find the quantity (in litres) of milk in the vessel at the end of this repetition. A. 72.9 B. 69 C. 70.2 D. 72 Status :   Unanswered Answer : A View Solution Quantity of milk in the vessel just after 9 liters of pure milk was withdrawn = (90-9) liters = 81 liters just after water was then added, the vessel would have had 81 liters of milk in 90 liters of mixtures. Therefore 9/10 the of its contents would have been milk been and 9 liters of the mixture was then withdrawn. Therefore just after the withdrawn 9/10(81) = 72.9 liters of milk remained in the vessel. Then water was added, this quantity of milk would be there in 90 liters of the mixture. Therefore required quantity = 72.9 liters Bookmark Share question feedback

Type: MCQ Single    |    Marks: +1/-0   |    Topic: Quant > Trigonometry    |    Time Spent:  0 Mins 41 Secs 6.  From the top of a building 30m high, the angle of elevation of the top of a tower is found to be equal to the angle of depression of the foot of the tower. Find the height of the tower. A. 15 m B. 30 m C. 60 m D. 90 m Status :   Unanswered Answer : C View Solution

Let AB=h and CD = x From \triangle

CDE△CDE

tan\theta =\frac { DE }{ CD }tanθ=CDDE tan\theta =\frac { 30 }{ x }tanθ=x30 x = \frac

{ 30 }{ tan\theta }tanθ30 =BE

From \triangle

ABE△ABE

tan\theta =\frac { AB }{ BE }tanθ=BEAB tan\theta =\frac { h }{ 30/tan\theta } \Rightarrow h=tan\theta \times 30/tan\thetatanθ=30/tanθh⇒h=tanθ×30/tanθ h=30m

Therefore height of the tower is AB + BC = 30 + 30 = 60 m Bookmark Share question feedback

Type: MCQ Single    |    Marks: +1/-0   |    Topic: Quant > Geometry    |    Time Spent:  1 Mins 58 Secs 7.  The perimeter of the triangle whose vertices are (3, 2), (7, 2) and (7, 5) (in units) is A. 5 B. 7 C. 9 D. 12 Status :   Unanswered Answer : D View Solution Perimeter = Sum of distance between the adjacent points

\sqrt { { 4 }^{ 2 }+{ 0 }^{ 2 }+ } \sqrt { { 0 }^{ 2 }+3^{ 2 }+ } \sqrt { { 4 } ^{ 2 }+{ 3 }^{ 2 } } =4+3+5=1242+02+02+32+42+32=4+3+5=12 Bookmark Share question feedback Type: MCQ Single    |    Marks: +1/-0   |    Topic: Quant > Equations    |    Time Spent:  0 Mins 29 Secs 8.  If the roots of 2x2 + 5x + 1 = 0 are each increased by 2, then the equation with these values as roots is A. 2x2+ 3x + 1 = 0 B. 2x2+ 3x - 1 = 0 C. 2x2- 3x + 1 = 0 D. 2x2- 3x - 1 = 0 Status :   Incorrect Given Answer : C Answer : D View Solution Required equation is obtained by replacing x by x -2 i.e., 2(x-2)2+5(x-2)+1=0 i.e., 2x2 - 3x-1=0 Bookmark Share question feedback Type: MCQ Single    |    Marks: +1/-0   |    Topic: Quant > Statistics    |    Time Spent:  1 Mins 2 Secs 9.  If the difference between the mode and the median is 16, then the difference between the mode and the mean is A. 20 B. 32 C. 33 D.  24

Status :   Incorrect Given Answer : B Answer : D View Solution

\frac { Mode-Median }{ Mode-Mean } =\frac { 2 }{ 3 }Mode−MeanMode−Median=32 \frac { 16 }{ x } =\frac { 2 }{ 3 } \Rightarrow x=\frac { 3 }{ 2 } \times 16=24x16=32⇒x=23×16=24 Bookmark Share question feedback Type: MCQ Single    |    Marks: +1/-0   |    Topic: Quant > Set Theory    |    Time Spent:  2 Mins 59 Secs 10.  If P(A)=0.32, P(B)=0.43 and P(A\cup B)=0.58,thenP(\overline { A } \cap B) (A∪B)=0.58,thenP(A∩B) = A. 0.15 B. 0.11 C. 0.26 D. 0.17 Status :   Incorrect Given Answer : D Answer : C View Solution P(A\cap B)=P(A)+P(B)-P(A\cup 0.58 = 0.17

B)(A∩B)=P(A)+P(B)−P(A∪B) = 0.32 + 0.43 -

P(\overline { A } \cap B)=P(B)-P(A\cap 0.26 Bookmark Share question feedback

B)(A∩B)=P(B)−P(A∩B) =0.43 - 0.17 =

Type: MCQ Single    |    Marks: +1/-0   |    Topic: Quant > Percentages    |    Time Spent:  1 Mins 9 Secs 11.  The salary of A is 25% more than the salary of B. By what percentage is the salary of B less than the salary of A? A. 20% B. 25% C. 30% D. 40% Status :   Correct

Answer : A View Solution Let the salary of B be Rs.100. As the salary of A is 25% more than that of B it will be Rs.125. The salary of B is less than the salary of A by \frac Bookmark Share question feedback

{ 25 }{ 125 }12525 x 100 i.e., 20%.

Type: MCQ Single    |    Marks: +1/-0   |    Topic: Quant > Percentages    |    Time Spent:  2 Mins 6 Secs 12.  Ajay and Vijay have some marbles with them. If 10% of the marbles with Ajay equals 30% of the marbles with Vijay and they have a total of 80 marbles, the number of marbles with Vijay is A. 10 B. 15 C. 20 D. 25 Status :   Unanswered Answer : C View Solution Let the number of marbles with Ajay and Vijay be A and V respectively. Given, \frac

{ 10 }{ 100 } A10010A = \frac { 30 }{ 100 } V10030V

=> A = 3V Also, A + V = 80 => 3V + V = 80 V = 20. Bookmark Share question feedback Type: MCQ Single    |    Marks: +1/-0   |    Topic: Quant > Ratio And Proportion    |   Time Spent:  0 Mins 55 Secs 13.  In a class the total number of boys and girls is 81. Which of the following can't be the ratio of the number of boys and girls? A. 1 : 2 B. 5 : 3 C. 7 : 2 D. 20 : 7 Status :   Incorrect Given Answer : D

Answer : B View Solution Going by the choices, the sum of the terms of the ratio in choice (2) is 8. But 81 is not, divisible by 8. So 5 : 3 is not possible. Bookmark Share question feedback Type: MCQ Single    |    Marks: +1/-0   |    Topic: Quant > Ratio And Proportion    |   Time Spent:  2 Mins 4 Secs 14.  The monthly expenses of A and B are in the ratio 4 : 3. Their monthly savings are in the ratio 3 : 2. Find the combined monthly savings of A and B, given, their incomes are र 10000 and र 7000 respectively. A. र 60,000 B. र 40,000 C. र 10,000 D. र 5,000 Status :   Correct Answer : C View Solution

Bookmark Share question feedback Type: MCQ Single    |    Marks: +1/-0   |    Topic: Quant > Mixtures And Alligations    |   Time Spent:  0 Mins 51 Secs 15.  In what ratio two milk solutions having 80% pure milk and 50% pure milk should be mixed such that the resultant solution will have 68% pure milk? A. 2 : 3 B. 3 : 2 C. 5 : 6 D. 6 : 5 Status :   Correct Answer : B View Solution

Bookmark Share question feedback Type: MCQ Single    |    Marks: +1/-0   |    Topic: Quant > Equations    |    Time Spent:  1 Mins 3 Secs 16.  What is the number of solutions for the equations 2x + 3y = 25 and 6x + 9y = 75? A. 0 B. 1 C. 2 D. infinite Status :   Incorrect Given Answer : A Answer : D View Solution Since the ratio of the coefficients of x in the first and the second equations is 1:3 and the ratio of the coefficients of y in the first and the second equation is also 1:3 the' set of equation will have infinite will have infinite solutions. Bookmark Share question feedback Passage :

Type: MCQ Single    |    Marks: +1/-0   |    Topic: Data Interpretation > Bar Graphs    |   Time Spent:  2 Mins 43 Secs 17.  Find the average annual growth rate in the exports over the given period approximately to two decimal places. A. 3.42% B. 2.38% C. 1.95% D. 2.96% Status :   Unanswered Answer : B View Solution

Bookmark Share question feedback Passage :

Type: MCQ Single    |    Marks: +1/-0   |    Topic: Data Interpretation > Bar Graphs    |   Time Spent:  0 Mins 57 Secs 18.  During which year was the ratio between the imports and the exports, the least? A. 2000 B. 2003 C. 2005 D. 2006 Status :   Incorrect Given Answer : A Answer : D View Solution From the given graph, it can be seen that imports is less than exports in 2000, 2003, 2005 and 2006 Among these years, 2006 has the least imports and the greatest exports. Least ratio will occur in 2006. Bookmark Share question feedback Passage :

Type: MCQ Single    |    Marks: +1/-0   |    Topic: Data Interpretation > Tables    |   Time Spent:  1 Mins 16 Secs 19.  Find the total number of females who died in accidents in the five districts together in 2008. A. 3673 B. 3579 C. 3484 D. 3797 Status :   Correct Answer : B View Solution The required total = 1004 + 1008 + 627 + 417 + 523 = 3579. Bookmark Share question feedback Passage :

Type: MCQ Single    |    Marks: +1/-0   |    Topic: Data Interpretation > Tables    |   Time Spent:  2 Mins 20 Secs 20.  The number of males who died due to accidents in district A in 2003 is approximately what percent of the total number of males who died due to accidents in district A over the given period? A. 14% B. 15% C. 11% D. 13% Status :   Incorrect Given Answer : B Answer : D View Solution The total number of males died in district A = 68 + 856 + 772 + 961 + 963 + 1168 = 5400 The required percentage =

\frac { 680 }{ 5400 } (100)5400680(100) % = \frac { 680 }{ 54 }54680 % = 13% Bookmark Share question feedback Passage :

Type: MCQ Single    |    Marks: +1/-0   |    Topic: Data Interpretation > Line Graphs    |   Time Spent:  0 Mins 36 Secs 21. 

In the year 2010, what percentage of the total number of employees in the public sector were employed in the state government? A. 21.37% B. 29.43% C. 42.37% D. 37.31% Status :   Unanswered Answer : D View Solution In the years 2010, the total number of employees = 3550 + 7275 + 6425 + 2250 = 19500 The required percentage = \frac

{ 7275 }{ 19500 }195007275 x 100

= 37.31% Bookmark Share question feedback Passage :

Type: MCQ Single    |    Marks: +1/-0   |    Topic: Data Interpretation > Line Graphs    |   Time Spent:  2 Mins 41 Secs 22.  What is the ratio of the total number of central and quasi government employees in 2009 to those working in local bodies and state government in 2011? A. 191 : 195 B. 191 : 197 C. 181 : 195 D. 171 : 185

Status :   Correct Answer : A View Solution The required ratio = (3350 + 6200) : (2300 + 7450) = 9550 : 9750 = 191 : 195 Bookmark Share question feedback Passage :

Type: MCQ Single    |    Marks: +1/-0   |    Topic: Data Interpretation > Line Graphs    |   Time Spent:  0 Mins 39 Secs 23.  During the year 2008, the number of employees of which of the following organisations is closest to the average number of employees per organisation in that year? A. Central government B. State government C. Quasi government D. Local bodies Status :   Incorrect Given Answer : D Answer : C View Solution average number of employees per organisation in 2008 = \frac

{ 3200+6450+4750+2200 }{ 4 }43200+6450+4750+2200 = 4150

4750 is the nearest value 4150. Bookmark Share question feedback Type: MCQ Single    |    Marks: +1/-0   |    Topic: Logical Reasoning > Directions    |   Time Spent:  1 Mins 40 Secs 24.  Starting from O, Vivek walked 40 m towards south, then he turned left and walked 60 m. He again turned left and walked 40 m. He once again turned left and walked 80 m and reached at D. How far and in which direction is D from O? A. 20 m, east B.  40 m, west C. 20 m, west D. 20 m, south Status :   Correct Answer : C View Solution

Bookmark Share question feedback Type: MCQ Single    |    Marks: +1/-0   |    Topic: Logical Reasoning > Clocks    |   Time Spent:  0 Mins 16 Secs 25.  A clock strikes once at 1O' clock, twice at 2O'clock, thrice at 3O' clock and so on. At 5O' clock if it takes 8 seconds to strike five times then, how long (in seconds) does it take to strike eleven times at 11'O clock? A. 22 B. 18 C. 20 D. 24 Status :   Unanswered Answer : C View Solution

Bookmark Share question feedback Type: MCQ Single    |    Marks: +1/-0   |    Topic: Logical Reasoning > Syllogism    |   Time Spent:  1 Mins 0 Secs 26.  The following question consists of three statements followed by two conclusions. Consider the statements to be true even if they vary from the commonly known facts and find out which of the conclusions logically follow/s the given statements and choose the proper alternative from the given choices. Statements: All pens are pencils. All pens are erasers. All erasers are chalks. Conclusions: I. All erasers are pens. II. Some erasers are pens. A. If only I follows. B. If only II follows. C. If neither I nor II follows. D. If both I and II follows. Status :   Incorrect Given Answer : C Answer : B View Solution

Bookmark Share question feedback

Passage : Directions for question: Select the correct alternative from the given choices. Type: MCQ Single    |    Marks: +1/-0   |    Topic: Verbal > Odd Sentence Out    |    Time Spent:  0 Mins 29 Secs 27.  Pick the odd one out. A. 145 B. 225 C. 197 D. 257 Status :   Unanswered Answer : B View Solution Bookmark Share question feedback Passage : Directions for question: Select the correct alternative from the given choices. Type: MCQ Single    |    Marks: +1/-0   |    Topic: Logical Reasoning > Calendars    |   Time Spent:  0 Mins 35 Secs 28.  For a car to Manufacture, it has to go through 3 steps in the following order. Step I: Designing, takes 19 days. Step II: Manufacturing, takes 24 days. Step III: Testing, takes 6 days. Only after completion of one step, the next step starts in the following day. If step II started on Friday, then on which day did step I start and step III end? A. Sunday, Saturday B. Thursday, Saturday C. Saturday, Sunday D. Saturday, Thursday Status :   Unanswered Answer : A View Solution

Bookmark Share question feedback

Type: MCQ Single    |    Marks: +1/-0   |    Topic: Logical Reasoning > Series    |    Time Spent:  0 Mins 20 Secs 29.  The value of the 12th term in the serves -1, -1-2, -1 -2 -3, -1 - 2 -3 -4, _______. A. -64 B. 96 C. -84 D. -78 Status :   Incorrect Given Answer : C Answer : D View Solution

Bookmark Share question feedback Type: MCQ Single    |    Marks: +1/-0   |    Topic: Logical Reasoning > Series    |    Time Spent:  1 Mins 6 Secs 30.  Find the next term in the series. RGHI, VKLM, ZOPQ, ______ A. BRSY B. CSTV C. DSRZ D. DSTU Status :   Correct Answer : D View Solution

Bookmark Share question feedback

Section Name : Verbal Ability And Reading Comprehension Passage : Directions for question: Identify the correct sentence or sentences: Type: MCQ Single    |    Marks: +1/-0   |    Topic: Verbal > Sentence Correction    |   Time Spent:  0 Mins 56 Secs 1.  (a) Organizational cultures take root in a myriad ways. (b) Some are shaped by a forceful leader. (c) Others take shape influenced by an industry ethic or a place. (d) Yet others take shape as a response to its environment. A. Only (a) B. Only (c) C. (b) and (c) D. (a) and (d) Status :   Correct Answer : C View Solution Sentence a must be either 'in myriad ways' or 'in a myriad of ways'. Sentence d must have 'their' (plural) not 'its' (singular) since the reference is to 'other'. b and c are' correct. Bookmark Share question feedback Passage : Directions for question: The given sentence has four underlined parts. One of them has a mistake. Mark the letter of the wrong part as the answer. Type: MCQ Single    |    Marks: +1/-0   |    Topic: Verbal > Sentence Correction    |   Time Spent:  1 Mins 4 Secs 2.  Young men and women (A) do not seem (B) to have a desire in sustaining (C) their marriage. (D) A. (A) B. (B) C. (C) D. (D) Status :   Incorrect Given Answer : B Answer : C View Solution The word 'desire' is followed by to + infinitive. Hence correction should be' .... seem to, have a desire to sustain their marriage'. Bookmark Share question feedback Type: MCQ Single    |    Marks: +1/-0   |    Topic: Verbal > Idioms And Phrases    |   Time Spent: 

0 Mins 45 Secs 3.  Directions for question: Every option has an idiom which is explained correctly in one of the four choices that follow. Pick out the correct choice. A.  to have a brush with the law - To encounter, or nearly encounter B.  in the bargain - to do bargaining C.  Fall by the wayside - getting into something D.  Burden of proof - to prove something Status :   Incorrect Given Answer : B Answer : A View Solution have a brush with (something) To encounter, or nearly encounter, someone or something. I had a brush with death when I was in that car accident. Oh, you'll have a brush with the law sooner or later if you keep robbing houses. Option A is the correct answer.   Bookmark Share question feedback Passage : Directions for question: Pick the most effective word from the given words to fill in the blanks to make the sentence meaningfully complete. Type: MCQ Single    |    Marks: +1/-0   |    Topic: Verbal > Fill In The Blanks    |    Time Spent:  0 Mins 39 Secs 4.  In ______ of her talents, many awards have come her way, but it is her ability to ______ a passion that she finds more rewarding. A. honour, produce B. view, create C. recognition, pursue D. anticipation, organize Status :   Correct Answer : C View Solution She was given many awards. This was done in 'recognition' of her talents, not in 'anticipation' of her talents. It can be to 'honour' her talents but not just in 'view' of, talents. But she finds 'pursuing' a passion to be more rewarding. Hence option (C). Bookmark Share question feedback

Passage : Directions for question: The question gives a word followed by four choices. From the choices, select the most suitable synonym (word which means the same) for the main word. Type: MCQ Single    |    Marks: +1/-0   |    Topic: Verbal > Synonyms    |    Time Spent:  0 Mins 17 Secs 5.  GROTESQUE A. bulky B. murky C. cumbersome D. weird Status :   Correct Answer : D View Solution 'Grotesque' means weird. Bookmark Share question feedback Passage : Directions for question: The question has a word followed by four choices. From the choices, identify the one which is opposite in meaning (antonym) to the main word. Type: MCQ Single    |    Marks: +1/-0   |    Topic: Verbal > Antonyms    |    Time Spent:  0 Mins 10 Secs 6.  FORTITUDE A. chivalry B. eruption C. cowardice D. chutzpah Status :   Correct Answer : C View Solution 'Fortitude' means courage. Hence its antonym is 'cowardice'. Bookmark Share question feedback Passage : Directions for question: Read the following sentence and choose the best replacement for the underlined part of the sentence from among the options given. Type: MCQ Single    |    Marks: +1/-0   |    Topic: Verbal > Sentence Improvement    |   Time Spent:  0 Mins 27 Secs 7.  The minister hit up at the persons who tried to malign his reputation. A. hit on

B. hit back C. hit around D. hit upon Status :   Correct Answer : B View Solution 'Hit back' is the most appropriate phrasal verb to be used here. To hit back is to attack, or criticize someone who has attacked or criticized you. Bookmark Share question feedback Passage : Directions for question: Match the idioms given in column A with correct meanings in column B. Type: MCQ Single    |    Marks: +1/-0   |    Topic: Verbal > Idioms And Phrases    |   Time Spent:  0 Mins 49 Secs 8. 

A. 1 - (a), 2 - (c), 3 - (d), 4 - (b) B. 1 - (b), 2 - (c), 3 - (a), 4 - (d) C. 1 - (a), 2 - (d), 3 - (b), 4 - (c) D. 1 - (b), 2 - (a), 3 - (d), 4 - (c) Status :   Incorrect Given Answer : A Answer : D View Solution Only (D) has the correct pairs. Bookmark Share question feedback Passage : Directions for question: Fill in the blank with the appropriate word. Type: MCQ Single    |    Marks: +1/-0   |    Topic: Verbal > Fill In The Blanks    |    Time Spent:  0 Mins 42 Secs 9.  The judicial ______ ordered by the State Government into the incidents should cover all aspects of the violence, including the complaints against some police personnel. A. study B. inquest

C. analysis D. probe Status :   Incorrect Given Answer : A Answer : D View Solution The judicial probe ( an investigation) ordered by the state government should cover all the aspects of violence and the complaints against some police personnel. The remaining choices are not suitable to the context. Study: Investigation and analysis of a subject analysis: a detailed examination. inquest: a judicial inquiry. In the context it is all investigation made by the police. Hence choice (D) is the appropriate answer. Bookmark Share question feedback Passage : Directions for question: Select the correct alternative from the given choices. Type: MCQ Single    |    Marks: +1/-0   |    Topic: Logical Reasoning > Statement Conclusion    |    Time Spent:  1 Mins 18 Secs 10.  A company reported a significant rise in its sales subsequent to changing the telecast time of its commercials from nonprime time to prime time. Hence it is concluded that for continued increase in sales the company must continue to arrange for telecasting the commercials during prime time even though it is costlier to do so.

Which of the following, if true, would most weaken the above conclusion? A. The difference between the costs of telecasting the commercials during prime and non prime time is only marginally less than the increased revenue due to increased sales. B. The company faced labour unrest for longer period during the time when the telecasting timings of the commercials were changed. C. The company had resorted to a different type of costing after it changed the telecast timings of the commercials. D. The chief competitor of the company closed down soon after timings were changed for the telecast of the commercials. Status :   Incorrect Given Answer : A Answer : D View Solution Change of telecast timing of commercials is given as the reasons for the increased sales. As (D) gives another possible explanation for the increase in sales. (D) is the correct answer. (A) is incorrect as the revenue is not the issue. (B) deals with labour

unrest hence supply so is irrelevant to sales. As cost is not discussed (C) is wrong similarly the cost telecasting is also an irrelevant issue. Bookmark Share question feedback Passage : Directions for question: Fill in the blank with the appropriate word. Type: MCQ Single    |    Marks: +1/-0   |    Topic: Verbal > Fill In The Blanks    |    Time Spent:  0 Mins 32 Secs 11.  The old city area in Hyderabad is known to be prone to communal violence, but, despite the early warnings and slow beginnings of the trouble, the police were found wanting in their response to a ______ situation, A. volatile B. mercurial C. capricious D. erratic Status :   Correct Answer : A View Solution The old city area in Hyderabad is known to be prone to communal violence but the police were found lacking in their response to a volatile situation. (i.e., highly unpredictable situation which is liable to change especially for the worse). The remaining choices are not appropriate mercurial: change in mood of a person. capricious: unaccountable changes of mood or behavior. erratic: irregular in pattern. Bookmark Share question feedback Passage : Directions for question: Read the following sentence and from among the options choose the best replacement for the underlined part of the sentence: Type: MCQ Single    |    Marks: +1/-0   |    Topic: Verbal > Sentence Improvement    |   Time Spent:  0 Mins 31 Secs 12.  The businessman was so optimistic that his losses would be recouped that he did not let them cast him away. A. cast him up B. cast him about C. cast him down D. cast him in Status :   Correct Answer : C View Solution

Option(C) is the right choice. Cast means to feel depressed. Cast away means to be stranded after a shipwreck, cast about means to search far and wide They do not fit here., cast in does not exist. Bookmark Share question feedback Passage : Directions for question: Read the following sentence and from among the options select the best replacement for the underlined part of the sentence: Type: MCQ Single    |    Marks: +1/-0   |    Topic: Verbal > Sentence Correction    |   Time Spent:  0 Mins 49 Secs 13.  When the little boy lost consciousness after a fall and was admitted to hospital, the doctor said he would come along with in two days. A. come up B. come out C. come back D. come round Status :   Incorrect Given Answer : B Answer : D View Solution Option (D) is the right choice come round is to regain consciousness. Come along is to progress, to appear come up is to arise come out is to be known to reveal and come back means to return. Only (D) fits the context. Bookmark Share question feedback Passage : Directions for question: The following sentence has four parts. Identify the part that has a grammatical error and mark that letter as your answer. Type: MCQ Single    |    Marks: +1/-0   |    Topic: Verbal > Sentence Correction    |   Time Spent:  0 Mins 43 Secs 14.  Psychological theories treat magic (A) / as a personal phenomena intended to meet (B) / individual needs, as opposed to (C) / social needs serving a collective purpose (D). A. A B. B C. C D. D Status :   Incorrect Given Answer : A Answer : B View Solution Option (B) is erroneous. Phenomenon is singular and agrees with 'a'. Phenomena is the plural form.

Bookmark Share question feedback Passage : Directions for question: Fill in the blank with the appropriate word. Type: MCQ Single    |    Marks: +1/-0   |    Topic: Verbal > Fill In The Blanks    |    Time Spent:  0 Mins 40 Secs 15.  A trivial dispute over a soiled currency note took a ______ proportion. A. menacing B. warning C. mitigating D. assuaging Status :   Incorrect Given Answer : B Answer : A View Solution Menacing is the most appropriate word it means (threating, frightening, intimidating)' i.e., according to the context we can understand that a trivial dispute has led to frightening consequences. Hence choice (a) the remaining words are not suitable to the context. warning : a caution mitigating : make less severe assuaging : make less intense. Bookmark Share question feedback Passage : Directions for question: Fill in the blank with the appropriate word. Type: MCQ Single    |    Marks: +1/-0   |    Topic: Verbal > Fill In The Blanks    |    Time Spent:  0 Mins 31 Secs 16.  That it ended in murder and arson is indicative of the deep ______ divide in Hyderabad. A. religious B. communal C. sectorial D. comical Status :   Incorrect Given Answer : A Answer : B View Solution The trivial issue has ended in a murder and it is indicative of the communal (conflict between different communities, or different religions of ethic origins) divide in Hyderabad. The remaining choices are not suitable to the context. sectorials Oflike a, sector religious : concerned with or believing a religion. comical : amusing.

Bookmark Share question feedback Passage : Directions for question: Fill in the blank with the appropriate word. Type: MCQ Single    |    Marks: +1/-0   |    Topic: Verbal > Fill In The Blanks    |    Time Spent:  0 Mins 15 Secs 17.  Hyderabad has a long history of politics based on religious ______. A. mechanisation B. discrimination C. polarisation D. dramatisation Status :   Incorrect Given Answer : B Answer : C View Solution Hyderabad has a long history of politics based on religious polarisation (divide into two sharply contrasting groups). The remaining choices are not suitable to the context mechanisation : automatic devices. discrimination : recognition and understanding the difference between two things. dramatisation : exaggeration. Bookmark Share question feedback Passage : Directions for question: Read the sentence to find out whether there is any error in it. The error if any will be in one part of the sentence. The number of that part is the answer. Type: MCQ Single    |    Marks: +1/-0   |    Topic: Verbal > Sentence Correction    |   Time Spent:  0 Mins 38 Secs 18.  The Indian IT industry (A) / is (B) / coping up (C) / with a lot of problems these days. (D) A. A B. B C. C D. D Status :   Incorrect Given Answer : D Answer : C View Solution The error is in part (C). The use of preposition 'up' with cope is incorrect. Bookmark Share question feedback

Type: MCQ Single    |    Marks: +1/-0   |    Topic: Verbal > Fill In The Blanks    |    Time Spent:  0 Mins 23 Secs 19.  Directions for question: Pick the most effective word from the given words to fill in the blanks to make the sentence meaningfully complete. Apart from having good _______ skills, the speaker had a _______ voice, which made the people listen to him with rapt attention. A. oratory, sonorous B. speaking, gruff C. lecturing, squeaky D. preaching, deep Status :   Incorrect Given Answer : B Answer : A View Solution The person mentioned is a 'speaker' or an 'orator'. Hence it is understandable that he will have 'oratory' skills. Also people will listen with rapt attention if the lecture is good and if the voice of the speaker is 'sonorous' i.e. resonant and impressive. A 'gruff',' 'squeaky' or 'deep' voice will not make people listen with rapt attention and hence options (B), (C) and (D) can be ruled out. Bookmark Share question feedback Type: MCQ Single    |    Marks: +1/-0   |    Topic: Verbal > Fill In The Blanks    |    Time Spent:  0 Mins 28 Secs 20.  Directions for question: Pick the most effective word from the given words to fill in the blanks to make the sentence meaningfully complete. A man who has _______ worldly life is not easily motivated to laugh because he has willingly _______ himself from society. A. sacrificed, ostracized B. abandoned, exiled C. renounced, isolated D. forsaken, banished Status :   Correct Answer : C View Solution A person can willingly 'renounce', 'abandon', 'forsake' or 'sacrifice' worldly life, Any of the four words can be used for the fist blank, But a person cannot willingly 'banish' or 'exile' himself from society, He doesn't 'ostracize' or exclude himself from society, He, 'isolates' himself from society and that is why he is not easily motivated to behave like others do, Hence the most appropriate pair of words would be 'renounced' and "isolated', Bookmark Share question feedback

Passage : Ancient Egypt developed a large, varied and fruitful medical tradition. Herodotus described the Egyptians as "the healthiest of all men, next to the Libyans", because of the dry climate and the notable public health system that they possessed. According to him, "the practice of medicine is so specialized among them that each physician is a healer of one disease and no more." Although Egyptian medicine, to a good extent, dealt with the supernatural, it eventually developed a practical use in the fields of anatomy, public health, and clinical diagnostics. The Atharvaveda, a sacred text of Hinduism dating from the Early Iron Age, is one of the first Indian text dealing with medicine, like the medicine of the Ancient Near East based on concepts of the exorcism of demons and magic. The Atharvaveda also contain prescriptions of herbs for various ailments. The use of herbs to treat ailments would later form a large part of Ayurveda. Ayurveda, meaning the "complete knowledge for long life" is another medical system of India. Its two most famous texts belong to the schools of Charaka and Sushruta. The earliest foundations of Ayurveda were built on a synthesis of traditional herbal practices together with a massive addition of theoretical conceptualizations, new nosologies and new therapies dating from about 600 BCE onwards, and coming out of the communities of thinkers who included the Buddha and others. According to the compendium of Charaka, the Charakasamhita, health and disease are not predetermined and life may be prolonged by human effort. The compendium of Susruta, the Susrutasamhita defines the purpose of medicine to cure the diseases of the sick, protect the healthy, and to prolong life. Both these ancient compendia include details of the examination, diagnosis, treatment, and prognosis of numerous ailments. The Susrutasamhita is notable for describing procedures on various forms of surgery, including rhinoplasty, the repair of torn ear lobes, perineal lithotomy, cataract surgery, and several other excisions and other surgical procedures. Most remarkable is Sushruta's penchant for scientific classification: His medical treatise consists of 184 chapters, 1,120 conditions are listed, including injuries and illnesses relating to aging and mental illness. The Sushruta Samhita describe 125 surgical instrument, 300 surgical procedures and classifies human surgery in 8 categories. The Ayurveda classics mention eight branches of medicine: kayacikitsa (internal medicine), salyacikitsa (surgery including anatomy), salakyacikitsa (eye, ear, nose, and throat diseases), kaumarabhṛtya (pediatrics), bhutavidya (spirit medicine), and agada tantra (toxicology), rasayana (science of rejuvenation), and vajikaraṇa (Aphrodisiac). Apart from learning these, the student of Ayurveda was expected to know ten arts that were indispensable in the preparation and application of his medicines: distillation, operative skills, cooking, horticulture, metallurgy, sugar manufacture, pharmacy, analysis and separation of minerals, compounding of metals, and preparation of alkalis. The teaching of various subjects was done during the instruction of relevant clinical subjects. For example, teaching of anatomy was a part of the teaching of surgery, embryology was a part of training in pediatrics and obstetrics, and the knowledge of physiology and pathology was interwoven in the teaching of all the clinical disciplines. The normal length of the student's training appears to have been seven years. But the physician was to continue to learn. As an alternative form of medicine in India, Unani medicine got deep roots and royal patronage during medieval times. It progressed during Indian sultanate and Mughal periods. Unani medicine is very close to Ayurveda. Both are based on theory of the presence of the elements in the human body. According to followers of Unani medicine,

these elements are present in different fluids and their balance leads to health and their imbalance leads to illness. By the 18th century A.D., Sanskrit medical wisdom still dominated. Muslim rulers built large hospitals in 1595 in Hyderabad, and in Delhi in 1719, and numerous commentaries on ancient texts were written. Type: MCQ Single    |    Marks: +1/-0   |    Topic: Verbal > Reading Comprehension    |   Time Spent:  5 Mins 29 Secs 21.  What is the main idea of the passage? A.  The history of medicine in Ancient Egypt and India. B.  Difference between Unani medicine and Ayurveda. C.  The various books that are written are discussed above. D.  Comparison between the ancient and present medicines. Status :   Correct Answer : A View Solution Option (A) is correct because this can be inferred as the main idea from the above passage. Option (B) is incorrect because this is not the main idea of the passage but is a part of the passage. Option (C) is incorrect because no such description of the books has been done in the passage. Option (D) is incorrect because no comparison can be seen between the medicines.  Bookmark Share question feedback Passage : Ancient Egypt developed a large, varied and fruitful medical tradition. Herodotus described the Egyptians as "the healthiest of all men, next to the Libyans", because of the dry climate and the notable public health system that they possessed. According to him, "the practice of medicine is so specialized among them that each physician is a healer of one disease and no more." Although Egyptian medicine, to a good extent, dealt with the supernatural, it eventually developed a practical use in the fields of anatomy, public health, and clinical diagnostics. The Atharvaveda, a sacred text of Hinduism dating from the Early Iron Age, is one of the first Indian text dealing with medicine, like the medicine of the Ancient Near East based on concepts of the exorcism of demons and magic. The Atharvaveda also contain prescriptions of herbs for various ailments. The use of herbs to treat ailments would later form a large part of Ayurveda. Ayurveda, meaning the "complete knowledge for long life" is another medical system of India. Its two most famous texts belong to the schools of Charaka and Sushruta. The earliest foundations of Ayurveda were built on a synthesis of traditional herbal practices together with a massive addition of theoretical conceptualizations, new nosologies and new therapies dating from about 600 BCE onwards, and coming out of the communities of thinkers who included the Buddha and others.

According to the compendium of Charaka, the Charakasamhita, health and disease are not predetermined and life may be prolonged by human effort. The compendium of Susruta, the Susrutasamhita defines the purpose of medicine to cure the diseases of the sick, protect the healthy, and to prolong life. Both these ancient compendia include details of the examination, diagnosis, treatment, and prognosis of numerous ailments. The Susrutasamhita is notable for describing procedures on various forms of surgery, including rhinoplasty, the repair of torn ear lobes, perineal lithotomy, cataract surgery, and several other excisions and other surgical procedures. Most remarkable is Sushruta's penchant for scientific classification: His medical treatise consists of 184 chapters, 1,120 conditions are listed, including injuries and illnesses relating to aging and mental illness. The Sushruta Samhita describe 125 surgical instrument, 300 surgical procedures and classifies human surgery in 8 categories. The Ayurveda classics mention eight branches of medicine: kayacikitsa (internal medicine), salyacikitsa (surgery including anatomy), salakyacikitsa (eye, ear, nose, and throat diseases), kaumarabhṛtya (pediatrics), bhutavidya (spirit medicine), and agada tantra (toxicology), rasayana (science of rejuvenation), and vajikaraṇa (Aphrodisiac). Apart from learning these, the student of Ayurveda was expected to know ten arts that were indispensable in the preparation and application of his medicines: distillation, operative skills, cooking, horticulture, metallurgy, sugar manufacture, pharmacy, analysis and separation of minerals, compounding of metals, and preparation of alkalis. The teaching of various subjects was done during the instruction of relevant clinical subjects. For example, teaching of anatomy was a part of the teaching of surgery, embryology was a part of training in pediatrics and obstetrics, and the knowledge of physiology and pathology was interwoven in the teaching of all the clinical disciplines. The normal length of the student's training appears to have been seven years. But the physician was to continue to learn. As an alternative form of medicine in India, Unani medicine got deep roots and royal patronage during medieval times. It progressed during Indian sultanate and Mughal periods. Unani medicine is very close to Ayurveda. Both are based on theory of the presence of the elements in the human body. According to followers of Unani medicine, these elements are present in different fluids and their balance leads to health and their imbalance leads to illness. By the 18th century A.D., Sanskrit medical wisdom still dominated. Muslim rulers built large hospitals in 1595 in Hyderabad, and in Delhi in 1719, and numerous commentaries on ancient texts were written. Type: MCQ Single    |    Marks: +1/-0   |    Topic: Verbal > Reading Comprehension    |   Time Spent:  0 Mins 58 Secs 22.  Why does the author use the term “sacred” for Atharvaveda? A.  Because it is a holy book for medicines. B.  Because it is considered as the oldest and the only book dealing with medicine. C.  Because it is a blessed book for medicine. D.  Because it is a desecrated book considered by the ancient Indian. Status :  

Incorrect Given Answer : A Answer : B View Solution Option (B) is correct because it is mentioned in the above passage that “The Atharvaveda, a sacred text of Hinduism dating from the Early Iron Age, is one of the first Indian text dealing with medicine, like the medicine of the Ancient Near East based on concepts of the exorcism of demons and magic”. Option (A) is incorrect because it does not have the right reason. Option (C) is incorrect because it does not have the right meaning for the term used in the passage. Option (D) is incorrect because the word desecrated means dishonoured which is opposite meaning as per the term has used in the passage.  Bookmark Share question feedback Passage : Ancient Egypt developed a large, varied and fruitful medical tradition. Herodotus described the Egyptians as "the healthiest of all men, next to the Libyans", because of the dry climate and the notable public health system that they possessed. According to him, "the practice of medicine is so specialized among them that each physician is a healer of one disease and no more." Although Egyptian medicine, to a good extent, dealt with the supernatural, it eventually developed a practical use in the fields of anatomy, public health, and clinical diagnostics. The Atharvaveda, a sacred text of Hinduism dating from the Early Iron Age, is one of the first Indian text dealing with medicine, like the medicine of the Ancient Near East based on concepts of the exorcism of demons and magic. The Atharvaveda also contain prescriptions of herbs for various ailments. The use of herbs to treat ailments would later form a large part of Ayurveda. Ayurveda, meaning the "complete knowledge for long life" is another medical system of India. Its two most famous texts belong to the schools of Charaka and Sushruta. The earliest foundations of Ayurveda were built on a synthesis of traditional herbal practices together with a massive addition of theoretical conceptualizations, new nosologies and new therapies dating from about 600 BCE onwards, and coming out of the communities of thinkers who included the Buddha and others. According to the compendium of Charaka, the Charakasamhita, health and disease are not predetermined and life may be prolonged by human effort. The compendium of Susruta, the Susrutasamhita defines the purpose of medicine to cure the diseases of the sick, protect the healthy, and to prolong life. Both these ancient compendia include details of the examination, diagnosis, treatment, and prognosis of numerous ailments. The Susrutasamhita is notable for describing procedures on various forms of surgery, including rhinoplasty, the repair of torn ear lobes, perineal lithotomy, cataract surgery, and several other excisions and other surgical procedures. Most remarkable is Sushruta's penchant for scientific classification: His medical treatise consists of 184 chapters, 1,120 conditions are listed, including injuries and illnesses relating to aging and mental illness. The Sushruta Samhita describe 125 surgical instrument, 300 surgical procedures and classifies human surgery in 8 categories. The Ayurveda classics mention eight branches of medicine: kayacikitsa (internal medicine), salyacikitsa (surgery including anatomy), salakyacikitsa (eye, ear, nose, and throat diseases), kaumarabhṛtya (pediatrics), bhutavidya (spirit medicine), and agada

tantra (toxicology), rasayana (science of rejuvenation), and vajikaraṇa (Aphrodisiac). Apart from learning these, the student of Ayurveda was expected to know ten arts that were indispensable in the preparation and application of his medicines: distillation, operative skills, cooking, horticulture, metallurgy, sugar manufacture, pharmacy, analysis and separation of minerals, compounding of metals, and preparation of alkalis. The teaching of various subjects was done during the instruction of relevant clinical subjects. For example, teaching of anatomy was a part of the teaching of surgery, embryology was a part of training in pediatrics and obstetrics, and the knowledge of physiology and pathology was interwoven in the teaching of all the clinical disciplines. The normal length of the student's training appears to have been seven years. But the physician was to continue to learn. As an alternative form of medicine in India, Unani medicine got deep roots and royal patronage during medieval times. It progressed during Indian sultanate and Mughal periods. Unani medicine is very close to Ayurveda. Both are based on theory of the presence of the elements in the human body. According to followers of Unani medicine, these elements are present in different fluids and their balance leads to health and their imbalance leads to illness. By the 18th century A.D., Sanskrit medical wisdom still dominated. Muslim rulers built large hospitals in 1595 in Hyderabad, and in Delhi in 1719, and numerous commentaries on ancient texts were written. Type: MCQ Single    |    Marks: +1/-0   |    Topic: Verbal > Reading Comprehension    |   Time Spent:  1 Mins 23 Secs 23.  Which of the following shows why the Ayurveda has complete knowledge? A.  By the 18th century A.D. Sanskrit medical wisdom still dominated. B.  As it is originally from India hence it has complete knowledge from the Vedas. C.  The student has to learn all the eight branches as well as ten arts that were indispensable in the preparation and application of his medicines. D.  The compendium of Susruta defines the purpose of medicine to cure the diseases of the sick, protect the healthy, and to prolong life. Status :   Incorrect Given Answer : A Answer : C View Solution Option (C) is correct as the author has mentioned above that the students need to have a complete knowledge about the illness as well as the medicines to make and even apply them. Option (A) is incorrect because it does not shows any relation with the question. Option (B) is incorrect because the very first line mentions that “Ancient Egypt developed a large, varied and fruitful medical tradition”. Option (D) is incorrect because this option describes a term Susruta.  Bookmark Share question feedback Passage :

Ancient Egypt developed a large, varied and fruitful medical tradition. Herodotus described the Egyptians as "the healthiest of all men, next to the Libyans", because of the dry climate and the notable public health system that they possessed. According to him, "the practice of medicine is so specialized among them that each physician is a healer of one disease and no more." Although Egyptian medicine, to a good extent, dealt with the supernatural, it eventually developed a practical use in the fields of anatomy, public health, and clinical diagnostics. The Atharvaveda, a sacred text of Hinduism dating from the Early Iron Age, is one of the first Indian text dealing with medicine, like the medicine of the Ancient Near East based on concepts of the exorcism of demons and magic. The Atharvaveda also contain prescriptions of herbs for various ailments. The use of herbs to treat ailments would later form a large part of Ayurveda. Ayurveda, meaning the "complete knowledge for long life" is another medical system of India. Its two most famous texts belong to the schools of Charaka and Sushruta. The earliest foundations of Ayurveda were built on a synthesis of traditional herbal practices together with a massive addition of theoretical conceptualizations, new nosologies and new therapies dating from about 600 BCE onwards, and coming out of the communities of thinkers who included the Buddha and others. According to the compendium of Charaka, the Charakasamhita, health and disease are not predetermined and life may be prolonged by human effort. The compendium of Susruta, the Susrutasamhita defines the purpose of medicine to cure the diseases of the sick, protect the healthy, and to prolong life. Both these ancient compendia include details of the examination, diagnosis, treatment, and prognosis of numerous ailments. The Susrutasamhita is notable for describing procedures on various forms of surgery, including rhinoplasty, the repair of torn ear lobes, perineal lithotomy, cataract surgery, and several other excisions and other surgical procedures. Most remarkable is Sushruta's penchant for scientific classification: His medical treatise consists of 184 chapters, 1,120 conditions are listed, including injuries and illnesses relating to aging and mental illness. The Sushruta Samhita describe 125 surgical instrument, 300 surgical procedures and classifies human surgery in 8 categories. The Ayurveda classics mention eight branches of medicine: kayacikitsa (internal medicine), salyacikitsa (surgery including anatomy), salakyacikitsa (eye, ear, nose, and throat diseases), kaumarabhṛtya (pediatrics), bhutavidya (spirit medicine), and agada tantra (toxicology), rasayana (science of rejuvenation), and vajikaraṇa (Aphrodisiac). Apart from learning these, the student of Ayurveda was expected to know ten arts that were indispensable in the preparation and application of his medicines: distillation, operative skills, cooking, horticulture, metallurgy, sugar manufacture, pharmacy, analysis and separation of minerals, compounding of metals, and preparation of alkalis. The teaching of various subjects was done during the instruction of relevant clinical subjects. For example, teaching of anatomy was a part of the teaching of surgery, embryology was a part of training in pediatrics and obstetrics, and the knowledge of physiology and pathology was interwoven in the teaching of all the clinical disciplines. The normal length of the student's training appears to have been seven years. But the physician was to continue to learn. As an alternative form of medicine in India, Unani medicine got deep roots and royal patronage during medieval times. It progressed during Indian sultanate and Mughal periods. Unani medicine is very close to Ayurveda. Both are based on theory of the presence of the elements in the human body. According to followers of Unani medicine,

these elements are present in different fluids and their balance leads to health and their imbalance leads to illness. By the 18th century A.D., Sanskrit medical wisdom still dominated. Muslim rulers built large hospitals in 1595 in Hyderabad, and in Delhi in 1719, and numerous commentaries on ancient texts were written. Type: MCQ Single    |    Marks: +1/-0   |    Topic: Verbal > Reading Comprehension    |   Time Spent:  0 Mins 32 Secs 24.  The author is likely to agree with which of the following questions? A.  The Ayurveda has its origin from the Ancient Egypt. B.  The Ayurveda is losing its importance in the present world. C.  Were the Unani medicine and royal patronage formed before Ayurveda’s? D.  Does the study of the Ayurveda take more than seven years? Status :   Correct Answer : D View Solution Option (D) is correct because it is mentioned in the above passage that “The normal length of the student's training appears to have been seven years. But the physician was to continue to learn”. Option (A) is incorrect because it can be inferred from the passage that Ayurveda has its roots in India. Option (B) is incorrect because no information can be seen in the passage hence the author cannot agree with the option. Option (C) is incorrect because it is mentioned in the passage that “Unani medicine is very close to Ayurveda” which shows that they are not formed before Ayurveda’s.  Bookmark Share question feedback Passage : Ancient Egypt developed a large, varied and fruitful medical tradition. Herodotus described the Egyptians as "the healthiest of all men, next to the Libyans", because of the dry climate and the notable public health system that they possessed. According to him, "the practice of medicine is so specialized among them that each physician is a healer of one disease and no more." Although Egyptian medicine, to a good extent, dealt with the supernatural, it eventually developed a practical use in the fields of anatomy, public health, and clinical diagnostics. The Atharvaveda, a sacred text of Hinduism dating from the Early Iron Age, is one of the first Indian text dealing with medicine, like the medicine of the Ancient Near East based on concepts of the exorcism of demons and magic. The Atharvaveda also contain prescriptions of herbs for various ailments. The use of herbs to treat ailments would later form a large part of Ayurveda. Ayurveda, meaning the "complete knowledge for long life" is another medical system of India. Its two most famous texts belong to the schools of Charaka and Sushruta. The earliest foundations of Ayurveda were built on a synthesis of traditional herbal practices together with a massive addition of theoretical conceptualizations, new nosologies and

new therapies dating from about 600 BCE onwards, and coming out of the communities of thinkers who included the Buddha and others. According to the compendium of Charaka, the Charakasamhita, health and disease are not predetermined and life may be prolonged by human effort. The compendium of Susruta, the Susrutasamhita defines the purpose of medicine to cure the diseases of the sick, protect the healthy, and to prolong life. Both these ancient compendia include details of the examination, diagnosis, treatment, and prognosis of numerous ailments. The Susrutasamhita is notable for describing procedures on various forms of surgery, including rhinoplasty, the repair of torn ear lobes, perineal lithotomy, cataract surgery, and several other excisions and other surgical procedures. Most remarkable is Sushruta's penchant for scientific classification: His medical treatise consists of 184 chapters, 1,120 conditions are listed, including injuries and illnesses relating to aging and mental illness. The Sushruta Samhita describe 125 surgical instrument, 300 surgical procedures and classifies human surgery in 8 categories. The Ayurveda classics mention eight branches of medicine: kayacikitsa (internal medicine), salyacikitsa (surgery including anatomy), salakyacikitsa (eye, ear, nose, and throat diseases), kaumarabhṛtya (pediatrics), bhutavidya (spirit medicine), and agada tantra (toxicology), rasayana (science of rejuvenation), and vajikaraṇa (Aphrodisiac). Apart from learning these, the student of Ayurveda was expected to know ten arts that were indispensable in the preparation and application of his medicines: distillation, operative skills, cooking, horticulture, metallurgy, sugar manufacture, pharmacy, analysis and separation of minerals, compounding of metals, and preparation of alkalis. The teaching of various subjects was done during the instruction of relevant clinical subjects. For example, teaching of anatomy was a part of the teaching of surgery, embryology was a part of training in pediatrics and obstetrics, and the knowledge of physiology and pathology was interwoven in the teaching of all the clinical disciplines. The normal length of the student's training appears to have been seven years. But the physician was to continue to learn. As an alternative form of medicine in India, Unani medicine got deep roots and royal patronage during medieval times. It progressed during Indian sultanate and Mughal periods. Unani medicine is very close to Ayurveda. Both are based on theory of the presence of the elements in the human body. According to followers of Unani medicine, these elements are present in different fluids and their balance leads to health and their imbalance leads to illness. By the 18th century A.D., Sanskrit medical wisdom still dominated. Muslim rulers built large hospitals in 1595 in Hyderabad, and in Delhi in 1719, and numerous commentaries on ancient texts were written. Type: MCQ Single    |    Marks: +1/-0   |    Topic: Verbal > Reading Comprehension    |   Time Spent:  1 Mins 36 Secs 25.  Which of the following does the author NOT agree with? A.  Unani medicine is very close to Ayurveda. Both are based on theory of the presence of the elements in the human body. B.  The normal length of the student's training appears to have been seven years. C. 

Medical treatise consists of 184 chapters, 1,120 conditions are listed, including injuries and illnesses relating to aging and mental illness. D.  The Atharvaveda, are not the one of the first Indian text dealing with medicine. Status :   Incorrect Given Answer : B Answer : D View Solution Option (D) is correct because it is mentioned in the passage that “The Atharvaveda, a sacred text of Hinduism dating from the Early Iron Age, is one of the first Indian text dealing with medicine, as the medicine of the Ancient Near East based on concepts of the exorcism of demons and magic”. Option (A) is incorrect because it is already mentioned in the passage hence the author will agree with the option. Option (B) is incorrect because the information is provided by the author in the passage. Option (C) is incorrect because it is already mentioned in the passage hence the author does not have any reason to disagree with the option.  Bookmark Share question feedback Passage : In Europe, before the Enlightenment of the eighteenth and nineteenth century, education was the responsibility of parents and the church. With the French and American Revolution education was established also as a public function. It was thought that the state, by assuming a more active role in the sphere of education, could help to make education available and accessible to all. Education had thus far been primarily available to the upper social classes and public education was perceived as a means of realising the egalitarian ideals underlining both revolutions. However, neither the American Declaration of Independence (1776) nor the French Declaration of the Rights of Man (1789) protected the right to education as the liberal concepts of human rights in the nineteenth century envisaged that parents retained the primary duty for providing education to their children. It was the states obligation to ensure that parents complied with this duty, and many states enacted legislation making school attendance compulsory. Furthermore, child labour laws were enacted to limit the number of hours per day children could be employed, to ensure children would attend school. States also became involved in the legal regulation of curricula and established minimum educational standards. In On Liberty John Stuart Mill wrote that an "education established and controlled by the State should only exist, if it exists at all, as one among many competing experiments, carried on for the purpose of example and stimulus to keep the others up to a certain standard of excellence." Liberal thinkers of the nineteenth century pointed to the dangers to too much state involvement in the sphere of education, but relied on state intervention to reduce the dominance of the church, and to protect the right to education of children against their own parents. In the latter half of the nineteenth century, educational rights were included in domestic bills of rights. The 1849 Paulskirchenverfassung, the constitution of the German Empire, strongly influenced subsequent European constitutions and devoted Article 152 to 158 of its bill of rights to education. The constitution recognised education as a function of the state, independent of the church. Remarkable at the time, the constitution proclaimed the right to free education for the poor, but the constitution did not explicitly require the state to set up educational

institutions. Instead the constitution protected the rights of citizens to found and operate schools and to provide home education. The constitution also provided for freedom of science and teaching, and it guaranteed the right of everybody to choose a vocation and train for it. The nineteenth century also saw the development of socialist theory, which held that the primary task of the state was to ensure the economic and social well-being of the community through government intervention and regulation. Socialist theory recognised that individuals had claims to basic welfare services against the state and education was viewed as one of these welfare entitlements. This was in contrast to liberal theory at the time, which regarded non-state actors as the prime providers of education. Socialist ideals were enshrined in the 1936 Soviet Constitution, which was the first constitution to recognise the right to education with a corresponding obligation of the state to provide such education. The constitution guaranteed free and compulsory education at all levels, a system of state scholarships and vocational training in state enterprises. Subsequently the right to education featured strongly in the constitutions of socialist states. As a political goal, right to education was declared in F. D. Roosevelt's 1944 speech on the Second Bill of Rights. Type: MCQ Single    |    Marks: +1/-0   |    Topic: Verbal > Reading Comprehension    |   Time Spent:  6 Mins 17 Secs 26.  What is the main idea of the passage? A.  To show how Right of education became a responsibility of each state. B.  To show diverse cultural and socio-economic factors. C.  To show the problems and causes for education. D.  Definition of Right to Education. Status :   Incorrect Given Answer : D Answer : A View Solution Option (A) is correct because the author in the above passage has mentioned that “It was the state's obligation to ensure that parents complied with this duty, and many states enacted legislation making school attendance compulsory”. Option (B) is incorrect because no information is provided in the passage about diverse cultural and socioeconomic factors. Option (C) is incorrect because no information about the problems and causes for education can be seen in the passage. Option (D) is incorrect because no definition can be seen in the passage.  Bookmark Share question feedback Passage : In Europe, before the Enlightenment of the eighteenth and nineteenth century, education was the responsibility of parents and the church. With the French and American Revolution education was established also as a public function. It was thought that the state, by assuming a more active role in the sphere of education, could help to make

education available and accessible to all. Education had thus far been primarily available to the upper social classes and public education was perceived as a means of realising the egalitarian ideals underlining both revolutions. However, neither the American Declaration of Independence (1776) nor the French Declaration of the Rights of Man (1789) protected the right to education as the liberal concepts of human rights in the nineteenth century envisaged that parents retained the primary duty for providing education to their children. It was the states obligation to ensure that parents complied with this duty, and many states enacted legislation making school attendance compulsory. Furthermore, child labour laws were enacted to limit the number of hours per day children could be employed, to ensure children would attend school. States also became involved in the legal regulation of curricula and established minimum educational standards. In On Liberty John Stuart Mill wrote that an "education established and controlled by the State should only exist, if it exists at all, as one among many competing experiments, carried on for the purpose of example and stimulus to keep the others up to a certain standard of excellence." Liberal thinkers of the nineteenth century pointed to the dangers to too much state involvement in the sphere of education, but relied on state intervention to reduce the dominance of the church, and to protect the right to education of children against their own parents. In the latter half of the nineteenth century, educational rights were included in domestic bills of rights. The 1849 Paulskirchenverfassung, the constitution of the German Empire, strongly influenced subsequent European constitutions and devoted Article 152 to 158 of its bill of rights to education. The constitution recognised education as a function of the state, independent of the church. Remarkable at the time, the constitution proclaimed the right to free education for the poor, but the constitution did not explicitly require the state to set up educational institutions. Instead the constitution protected the rights of citizens to found and operate schools and to provide home education. The constitution also provided for freedom of science and teaching, and it guaranteed the right of everybody to choose a vocation and train for it. The nineteenth century also saw the development of socialist theory, which held that the primary task of the state was to ensure the economic and social well-being of the community through government intervention and regulation. Socialist theory recognised that individuals had claims to basic welfare services against the state and education was viewed as one of these welfare entitlements. This was in contrast to liberal theory at the time, which regarded non-state actors as the prime providers of education. Socialist ideals were enshrined in the 1936 Soviet Constitution, which was the first constitution to recognise the right to education with a corresponding obligation of the state to provide such education. The constitution guaranteed free and compulsory education at all levels, a system of state scholarships and vocational training in state enterprises. Subsequently the right to education featured strongly in the constitutions of socialist states. As a political goal, right to education was declared in F. D. Roosevelt's 1944 speech on the Second Bill of Rights. Type: MCQ Single    |    Marks: +1/-0   |    Topic: Verbal > Reading Comprehension    |   Time Spent:  0 Mins 11 Secs 27.  Why does the author use the term “Enlightenment”? A.  it means obfuscation. B. 

it means education. C.  it means illumination. D.  it means ignorance. Status :   Correct Answer : B View Solution Option (B) is correct because it is mentioned rightly as per the passage. Option (A) is incorrect because the word obfuscation means complication and it is an opposite word for the term used. Option (C) is incorrect because the word illumination is used in terms of light which is not right for the term. Option (D) is incorrect because the word ignorance means unawareness which is a opposite word for the term.  Bookmark Share question feedback Passage : In Europe, before the Enlightenment of the eighteenth and nineteenth century, education was the responsibility of parents and the church. With the French and American Revolution education was established also as a public function. It was thought that the state, by assuming a more active role in the sphere of education, could help to make education available and accessible to all. Education had thus far been primarily available to the upper social classes and public education was perceived as a means of realising the egalitarian ideals underlining both revolutions. However, neither the American Declaration of Independence (1776) nor the French Declaration of the Rights of Man (1789) protected the right to education as the liberal concepts of human rights in the nineteenth century envisaged that parents retained the primary duty for providing education to their children. It was the states obligation to ensure that parents complied with this duty, and many states enacted legislation making school attendance compulsory. Furthermore, child labour laws were enacted to limit the number of hours per day children could be employed, to ensure children would attend school. States also became involved in the legal regulation of curricula and established minimum educational standards. In On Liberty John Stuart Mill wrote that an "education established and controlled by the State should only exist, if it exists at all, as one among many competing experiments, carried on for the purpose of example and stimulus to keep the others up to a certain standard of excellence." Liberal thinkers of the nineteenth century pointed to the dangers to too much state involvement in the sphere of education, but relied on state intervention to reduce the dominance of the church, and to protect the right to education of children against their own parents. In the latter half of the nineteenth century, educational rights were included in domestic bills of rights. The 1849 Paulskirchenverfassung, the constitution of the German Empire, strongly influenced subsequent European constitutions and devoted Article 152 to 158 of its bill of rights to education. The constitution recognised education as a function of the state, independent of the church. Remarkable at the time, the constitution proclaimed the right to free education for the poor, but the constitution did not explicitly require the state to set up educational institutions. Instead the constitution protected the rights of citizens to found and operate schools and to provide home education. The constitution also provided for freedom of

science and teaching, and it guaranteed the right of everybody to choose a vocation and train for it. The nineteenth century also saw the development of socialist theory, which held that the primary task of the state was to ensure the economic and social well-being of the community through government intervention and regulation. Socialist theory recognised that individuals had claims to basic welfare services against the state and education was viewed as one of these welfare entitlements. This was in contrast to liberal theory at the time, which regarded non-state actors as the prime providers of education. Socialist ideals were enshrined in the 1936 Soviet Constitution, which was the first constitution to recognise the right to education with a corresponding obligation of the state to provide such education. The constitution guaranteed free and compulsory education at all levels, a system of state scholarships and vocational training in state enterprises. Subsequently the right to education featured strongly in the constitutions of socialist states. As a political goal, right to education was declared in F. D. Roosevelt's 1944 speech on the Second Bill of Rights. Type: MCQ Single    |    Marks: +1/-0   |    Topic: Verbal > Reading Comprehension    |   Time Spent:  0 Mins 21 Secs 28.  Which of the following exemplifies that socialist theory was important? A.  As education was the responsibility of parents and the church. B.  As it created awareness in the society about education. C.  As the economic and social well-being were taken care of by the government intervention and regulation. D.  It helped the secondary education also to be compulsory for the children. Status :   Correct Answer : C View Solution Option (C) is correct as it is mentioned in the passage that “The nineteenth century also saw the development of socialist theory, which held that the primary task of the state was to ensure the economic and social well-being of the community through government intervention and regulation”. Option (A) is incorrect because the option is not related to the question. Option (B) is incorrect because it did not create any awareness in the society. Option (D) is incorrect because no information about the secondary education is mentioned in the passage.  Bookmark Share question feedback Passage : In Europe, before the Enlightenment of the eighteenth and nineteenth century, education was the responsibility of parents and the church. With the French and American Revolution education was established also as a public function. It was thought that the state, by assuming a more active role in the sphere of education, could help to make education available and accessible to all. Education had thus far been primarily available

to the upper social classes and public education was perceived as a means of realising the egalitarian ideals underlining both revolutions. However, neither the American Declaration of Independence (1776) nor the French Declaration of the Rights of Man (1789) protected the right to education as the liberal concepts of human rights in the nineteenth century envisaged that parents retained the primary duty for providing education to their children. It was the states obligation to ensure that parents complied with this duty, and many states enacted legislation making school attendance compulsory. Furthermore, child labour laws were enacted to limit the number of hours per day children could be employed, to ensure children would attend school. States also became involved in the legal regulation of curricula and established minimum educational standards. In On Liberty John Stuart Mill wrote that an "education established and controlled by the State should only exist, if it exists at all, as one among many competing experiments, carried on for the purpose of example and stimulus to keep the others up to a certain standard of excellence." Liberal thinkers of the nineteenth century pointed to the dangers to too much state involvement in the sphere of education, but relied on state intervention to reduce the dominance of the church, and to protect the right to education of children against their own parents. In the latter half of the nineteenth century, educational rights were included in domestic bills of rights. The 1849 Paulskirchenverfassung, the constitution of the German Empire, strongly influenced subsequent European constitutions and devoted Article 152 to 158 of its bill of rights to education. The constitution recognised education as a function of the state, independent of the church. Remarkable at the time, the constitution proclaimed the right to free education for the poor, but the constitution did not explicitly require the state to set up educational institutions. Instead the constitution protected the rights of citizens to found and operate schools and to provide home education. The constitution also provided for freedom of science and teaching, and it guaranteed the right of everybody to choose a vocation and train for it. The nineteenth century also saw the development of socialist theory, which held that the primary task of the state was to ensure the economic and social well-being of the community through government intervention and regulation. Socialist theory recognised that individuals had claims to basic welfare services against the state and education was viewed as one of these welfare entitlements. This was in contrast to liberal theory at the time, which regarded non-state actors as the prime providers of education. Socialist ideals were enshrined in the 1936 Soviet Constitution, which was the first constitution to recognise the right to education with a corresponding obligation of the state to provide such education. The constitution guaranteed free and compulsory education at all levels, a system of state scholarships and vocational training in state enterprises. Subsequently the right to education featured strongly in the constitutions of socialist states. As a political goal, right to education was declared in F. D. Roosevelt's 1944 speech on the Second Bill of Rights. Type: MCQ Single    |    Marks: +1/-0   |    Topic: Verbal > Reading Comprehension    |   Time Spent:  0 Mins 28 Secs 29.  The author is likely to agree with which of the following questions? A.  Is the right to education a universal entitlement to education? B. 

Is the right to education recognized in the International Covenant on Economic, Social and Cultural Rights as a human right? C.  Will the realisation of the right to education on a national level be achieved through compulsory education only? D.  Was the right to education declared in F. D. Roosevelt's 1944 speech on the Second Bill of Rights? Status :   Correct Answer : D View Solution Option (D) is correct because it is mentioned in the last line of the passage that “As a political goal, right to education was declared in F. D. Roosevelt's 1944 speech on the Second Bill of Rights”. Option (A) is incorrect because is not mentioned in the passage hence the author will not agree with option. Option (B) is incorrect because no information is provided in the passage. Option (C) is incorrect because this is not mentioned in the passage hence the author will not agree with option.  Bookmark Share question feedback Passage : In Europe, before the Enlightenment of the eighteenth and nineteenth century, education was the responsibility of parents and the church. With the French and American Revolution education was established also as a public function. It was thought that the state, by assuming a more active role in the sphere of education, could help to make education available and accessible to all. Education had thus far been primarily available to the upper social classes and public education was perceived as a means of realising the egalitarian ideals underlining both revolutions. However, neither the American Declaration of Independence (1776) nor the French Declaration of the Rights of Man (1789) protected the right to education as the liberal concepts of human rights in the nineteenth century envisaged that parents retained the primary duty for providing education to their children. It was the states obligation to ensure that parents complied with this duty, and many states enacted legislation making school attendance compulsory. Furthermore, child labour laws were enacted to limit the number of hours per day children could be employed, to ensure children would attend school. States also became involved in the legal regulation of curricula and established minimum educational standards. In On Liberty John Stuart Mill wrote that an "education established and controlled by the State should only exist, if it exists at all, as one among many competing experiments, carried on for the purpose of example and stimulus to keep the others up to a certain standard of excellence." Liberal thinkers of the nineteenth century pointed to the dangers to too much state involvement in the sphere of education, but relied on state intervention to reduce the dominance of the church, and to protect the right to education of children against their own parents. In the latter half of the nineteenth century, educational rights were included in domestic bills of rights. The 1849 Paulskirchenverfassung, the constitution of the German Empire, strongly influenced subsequent European constitutions and devoted Article 152 to 158 of its bill of rights to education. The constitution recognised education as a function of the state, independent of the church. Remarkable at the time, the constitution proclaimed the right to free education for the

poor, but the constitution did not explicitly require the state to set up educational institutions. Instead the constitution protected the rights of citizens to found and operate schools and to provide home education. The constitution also provided for freedom of science and teaching, and it guaranteed the right of everybody to choose a vocation and train for it. The nineteenth century also saw the development of socialist theory, which held that the primary task of the state was to ensure the economic and social well-being of the community through government intervention and regulation. Socialist theory recognised that individuals had claims to basic welfare services against the state and education was viewed as one of these welfare entitlements. This was in contrast to liberal theory at the time, which regarded non-state actors as the prime providers of education. Socialist ideals were enshrined in the 1936 Soviet Constitution, which was the first constitution to recognise the right to education with a corresponding obligation of the state to provide such education. The constitution guaranteed free and compulsory education at all levels, a system of state scholarships and vocational training in state enterprises. Subsequently the right to education featured strongly in the constitutions of socialist states. As a political goal, right to education was declared in F. D. Roosevelt's 1944 speech on the Second Bill of Rights. Type: MCQ Single    |    Marks: +1/-0   |    Topic: Verbal > Reading Comprehension    |   Time Spent:  0 Mins 40 Secs 30.  Which of the following does the author NOT agree with? A.  The constitution guaranteed free and compulsory education at all levels. B.  Socialist ideals were enshrined in the 1936 Soviet Constitution, which was the first constitution to recognise the right to education. C.  The constitution recognised education as a function of the state, independent of the church. D.  International law protects the right to pre-primary education in a state. Status :   Incorrect Given Answer : C Answer : D View Solution Option (D) is correct because the international laws cannot be applicable in different states. The international laws are made on general bases. Option (A) is incorrect because it is already mentioned in the passage that “The constitution guaranteed free and compulsory education at all levels, a system of state scholarships and vocational training in state enterprises”. Option (B) is incorrect because it is already mentioned in the passage that “Socialist ideals were enshrined in the 1936 Soviet Constitution, which was the first constitution to recognise the right to education with a corresponding obligation of the state to provide such education”. Option (C) is incorrect because it is already mentioned in the passage that “The 1849 Paulskirchenverfassung, the constitution of the German Empire, strongly influenced subsequent European constitutions and devoted Article 152 to 158 of its bill of rights to education. The constitution recognised education as a function of the state, independent of the church”. 

Bookmark Share question feedback

Section Name : General Awareness Type: MCQ Single    |    Marks: +1/-0   |    Topic: General Awareness > Static    |   Time Spent:  0 Mins 13 Secs 1.  The annulment of partition of Bengal was done by which of the following? A.  Lord Hardinge B.  Lord Willian Bentinck C.  Lord Canning D.  Lord Lytton Status :   Incorrect Given Answer : D Answer : A View Solution Lord Hardinge, the Viceroy of India from 1910 to 1916, had annulled the partition of Bengal in 1911.  The partition of Bengal was done by Lord Curzon on 16th October and the Annullment bof Bengal was done by Lord Hardinge in 1911. Bookmark Share question feedback Type: MCQ Single    |    Marks: +1/-0   |    Topic: General Awareness > Static    |   Time Spent:  0 Mins 14 Secs 2.  Which of the following newspapers is NOT related to 'Madan Mohan Malaviya' ?  A.  Leader B.  Comrade C.  Hindustan D.  Abhyudaya Status :   Incorrect Given Answer : C Answer : B View Solution

Comrade is not related to Madan Mohan Malaviya who had been served as the president of Indian National Confress for four times. Comrade was published and edited by Maulana Mohammad Ali between 1911 and 1914. Bookmark Share question feedback Type: MCQ Single    |    Marks: +1/-0   |    Topic: General Awareness > Static    |   Time Spent:  0 Mins 11 Secs 3.  Which state is the largest producer of MICA in india? A.  Odisha B.  Rajasthan C.  Jharkhand D.  Andhra Pradesh Status :   Incorrect Given Answer : C Answer : D View Solution Andhra Pradesh is the largest producer of MICA in India followed by Rajasthan and Jharkhand. MICA is used in paints. Bookmark Share question feedback Type: MCQ Single    |    Marks: +1/-0   |    Topic: General Awareness > Static    |   Time Spent:  0 Mins 16 Secs 4.  Which layer of the atmosphere has the maximum concentration of Ozone gas? A.  Troposphere B.  Stratosphere C.  Thermosphere D.  Mesosphere Status :   Correct Answer : B View Solution Stratosphere layer of atmosphere has the maximum concentration of Ozone gas. Approximately 10 per cent of Ozone layer is present in Troposphere and rest of the 90 per cent found in Stratosphere layer of atmosphere. Ozone layer in Stratosphere atmosphere absorbs most of the ultraviolet radiation from the Sun. Stratosphere is the

second layer of atmosphere whereas Troposphere is the lowest layer of the atmossphere. The Thermosphere is the layer of the Earth's atmosphere is above the Mesosphere. There are five layers of atmosphere namely - Troposphere, Stratosphere, Mesosphere, Thermosphere and Exosphere. Bookmark Share question feedback Type: MCQ Single    |    Marks: +1/-0   |    Topic: General Awareness > Static    |   Time Spent:  0 Mins 20 Secs 5.  Which of the following is the highest source of protein ? A.  Sun Flower B.  Soyabeans C.  Grams D.  Wheat Status :   Incorrect Given Answer : C Answer : B View Solution Soyabean has the highest source of protein content of 36 g, Grams contain 19g protein content, Sun Flower protein content is 21g and wheat is 13.2g Bookmark Share question feedback Type: MCQ Single    |    Marks: +1/-0   |    Topic: General Awareness > Static    |   Time Spent:  0 Mins 13 Secs 6.  Which country imposed the world's toughest law against plastic bags recently? A.  Japan  B.  India C.  Kenya D.  France Status :   Incorrect Given Answer : A Answer : C View Solution Kenya imposed the world's toughest law against plastic bags recently.

If they sell or use any plastic bags it would become a risk of imprisonment of upto four years or fines of $40,000.  Plastic is banned in more than 40 other countries including China, France, Rwanda and Italy. Bookmark Share question feedback Type: MCQ Single    |    Marks: +1/-0   |    Topic: General Awareness > Static    |   Time Spent:  0 Mins 11 Secs 7.  When was the Montreal protocol assigned to reduce production of Chlorofluorocarbons? A.  2001 B.  1980 C.  1987 D.  1994 Status :   Correct Answer : C View Solution Montreal Protocol is an international treaty to protect the Ozone layer. It was agreed on 26th August 1987. Its main objective was phasing out the production of numerous substances that are responsible for Ozone depletion. It came into effect on 1st January 1989. It became its signatory member on 19th June 1992.  Bookmark Share question feedback Type: MCQ Single    |    Marks: +1/-0   |    Topic: General Awareness > Static    |   Time Spent:  0 Mins 8 Secs 8.  Which of the following countries has built an 'artificial sun'? A.  Japan B.  China C.  Singapore D.  Israel Status :   Correct Answer : B View Solution

China has built an 'artificial sun' that reaches temperature six times that of the core of the sun. The reactor named 'Experimental Advanced Superconducting Tokamak' (EAST) is designed to replicate the processes of the sun as part of a project to turn hydrogen into cost- effective green energy, It involves fusion of hydrogen atoms together to form heavier elements, such as helium.  Bookmark Share question feedback Type: MCQ Single    |    Marks: +1/-0   |    Topic: General Awareness > Static    |   Time Spent:  0 Mins 13 Secs 9.  According to the Indus Water treaty which river will be not governed by Pakistan?   A.  Indus B.  Chenab C.  Jhelum D.  Beas Status :   Incorrect Given Answer : A Answer : D View Solution The Indus Water treaty (IWT) is a water distribution treaty between India and Pakistan which was signed on September 19, 1960. The treaty deals with river Indus and its five tributaries.These are divided into two categories -  Eastern Rivers: 1. 2. 3.

Sutlej Beas Ravi Western Rivers:

1. 2. 3.

Jhelum Chenab Indus According to treaty, all the water of eastern rivers shall be available for unrestricted use in India. India should let unrestricted flow of water from western rivers to Pakistan.  Bookmark Share question feedback

Type: MCQ Single    |    Marks: +1/-0   |    Topic: General Awareness > Static    |   Time Spent:  0 Mins 15 Secs 10.  Which one of the following is a Wartime Gallantry Award? A.  Shaurya Chakra  B.  Kirti Chakra C.  Yudh Seva Medal  D.  Param Vir Chakra Status :   Correct Answer : D View Solution Peacetime Gallantry Award1. 2. 3.

Ashok Chakra Award Kirti Chakra Shaurya Chakra Wartime Gallantry Award - 

1. 2. 3.

Param Vir Chakra Maha Vir Chakra Vir Chakra Wartime Distinguished Service -

1. 2. 3.

Sarvottam Yudh Seva Medal Uttam Yudh Seva Medal Yudh Seva Medal   Bookmark Share question feedback Type: MCQ Single    |    Marks: +1/-0   |    Topic: General Awareness > Static    |   Time Spent:  0 Mins 13 Secs 11.  'Kunitha' is a ritual dance form of which state?  A.  Gujarat B.  Bihar C. 

Karnataka D.  Madhya Pradesh  Status :   Correct Answer : C View Solution The ritual dances of Karnataka are known as Kunitha. One such dance is the Dollu Kunitha, a popular dance form accompanied by singing and the beats of decorated drums. This dance is primarily performed by men from the shepherd or Kuruba caste. The Dollu Kunitha is characterized by vigorous drumbeats, quick movements and synchronized group formations.  Bookmark Share question feedback Type: MCQ Single    |    Marks: +1/-0   |    Topic: General Awareness > Static    |   Time Spent:  0 Mins 13 Secs 12.  The second Ozone hole was detected over which of the following? A.  Antartica B.  Australia C.  Arctic D.  Sweden Status :   Incorrect Given Answer : A Answer : C View Solution The second Ozone hole was detected over Arctic. Bookmark Share question feedback Type: MCQ Single    |    Marks: +1/-0   |    Topic: General Awareness > Static    |   Time Spent:  0 Mins 12 Secs 13.  With which of the following is 'Van Mahotsav' associated? A.  Insurance of Trees B.  Genetic modification of Trees C.  Planting of Trees D.  Increase in Crops

Status :   Correct Answer : C View Solution Van Mohatsav is an annual tree- planting movement in India, which began in 1950. The name Van Mahotsav means Festival of Forest. It was started by K.M. Munshi. Bookmark Share question feedback Type: MCQ Single    |    Marks: +1/-0   |    Topic: General Awareness > Static    |   Time Spent:  0 Mins 13 Secs 14.  Who among the following is not a permanent member of Malabar trilateral naval exercise? A.  United States of America B.  India  C.  Japan  D.  Vietnam Status :   Incorrect Given Answer : B Answer : D View Solution Exercise Malabar is a trilateral naval exercise involving the United States of America, Japan and India as permanent partners. Originally begun in 1992 as a bilateral exercise between India and the United States. Japan became a permanent partner in 2015. Past non permanent participants were Australia and Singapore. Bookmark Share question feedback Type: MCQ Single    |    Marks: +1/-0   |    Topic: General Awareness > Static    |   Time Spent:  0 Mins 11 Secs 15.  Who among the following was a classical Indian flute player? A.  Bismillah Khan B.  Hariprasad Chaurasia C.  Zakir Hussain D.  Shiv Kumar Sharma Status :  

Incorrect Given Answer : A Answer : B View Solution

Artists

Instruments

Bismillah Khan

Shehnai

Hariprasad Chaurasia

Flute

Zakir Hussain

Tabla 

Shiv Kumar Sharma

Santoor

  Bookmark Share question feedback Type: MCQ Single    |    Marks: +1/-0   |    Topic: General Awareness > Static    |   Time Spent:  0 Mins 9 Secs 16.  Which of the following is capital of Cambodia? A.  Nicosia B.  Hanoi C.  Dili D.  Phnom Penh Status :   Incorrect Given Answer : B Answer : D View Solution

Capital of Cambodia

Phnom Penh

Currency

Cambodian Riel

King

Norodom Sihamoni

Prime Minister

Hun Sen

Official Language

Khmer

  Bookmark Share question feedback Type: MCQ Single    |    Marks: +1/-0   |    Topic: General Awareness > Static    |   Time Spent:  0 Mins 17 Secs

17.  Which one of the following Vedas dealt with rituals? A.  Rig Veda  B.  Sama Veda  C.  Yajur Veda D.  Atharva Veda Status :   Incorrect Given Answer : A Answer : C View Solution There are four Vedas : 1. 2. 3. 4.

Rig Veda  Sama Veda Yajur Veda Atharva Veda Rig Veda is a collection of hymns dedicated to various deities. The Sama Veda deals with melodies. The Yajur Veda contains the rituals of the Yagna. The Atharva Veda contains magic spells. Bookmark Share question feedback Type: MCQ Single    |    Marks: +1/-0   |    Topic: General Awareness > Static    |   Time Spent:  0 Mins 9 Secs 18.  How many schedules are there in our Constitution ? A.  10 B.  8 C.  12 D.  11 Status :   Correct Answer : C View Solution

The Indian Constitution has 12 schedules  and 448 articles  written in 25 parts. Also, 103 amendments have been made in it till now. It was also the world's lengthiest written constitution at the time of its commencement with 8 schedules and 395 articles in 22 parts. Bookmark Share question feedback Type: MCQ Single    |    Marks: +1/-0   |    Topic: General Awareness > Static    |   Time Spent:  0 Mins 19 Secs 19.  The Kishtwar National park (KNP) is located in which state?  A.  Jammu and Kashmir B.  Himachal Pradesh C.  Punjab D.  Sikkim Status :   Correct Answer : A View Solution Kishtwar National Park (KNP) is located in the Kishtwar district of Jammu and Kashmir, India. This national Park was made to protect the endangered species of snow leopards and was given a status of a national park in 1981. Bookmark Share question feedback Type: MCQ Single    |    Marks: +1/-0   |    Topic: General Awareness > Static    |   Time Spent:  0 Mins 10 Secs 20.  The concept of single citizenship in the Indian Constitution is inspired from which country?  A.  USA B.  France  C.  Ireland D.  England Status :   Incorrect Given Answer : C Answer : D View Solution

Though the Indian Constitution is federal and envisages dual polity, it provides for only single citizenship unlike USA that has dual citizenship. This feature has been adopted from British Constitution. Bookmark Share question feedback Type: MCQ Single    |    Marks: +1/-0   |    Topic: General Awareness > Static    |   Time Spent:  0 Mins 10 Secs 21.  What is the shape of the Atlantic Ocean? A.  Almost Circular B.  'S' Shaped  C.  Almost Triangular D.  None of these Status :   Incorrect Given Answer : D Answer : B View Solution The shape of the Atlantic Ocean is of an 'S' shaped basin. It extends between Europe and Africa to the east and the America's to the west. The Atlantic Ocean is the second largest ocean in the World after Pacific Ocean. Bookmark Share question feedback Type: MCQ Single    |    Marks: +1/-0   |    Topic: General Awareness > Static    |   Time Spent:  0 Mins 10 Secs 22.  Which country has withdrawn from UN-backed anti-corruption commission? A.  Ukraine B.  Uganda C.  Guatemala D.  Netherlands Status :   Incorrect Given Answer : B Answer : C View Solution

Guatemala has said it is withdrawing from a UN- backed anti corruption commission. President Jimmy Morales accused the International Commission Against Impunity in Guatemala, commonly known as CICIG, of polarising the country and putting its security at risk. Bookmark Share question feedback Type: MCQ Single    |    Marks: +1/-0   |    Topic: General Awareness > Static    |   Time Spent:  0 Mins 14 Secs 23.  Which of the following state has a maximum number of International Airports? A.  Madhya Pradesh B.  Karnataka C.  Tamil nadu  D.  Kerala Status :   Correct Answer : D View Solution Kerala has maximum 4 international Airports. 1. 2. 3. 4.

Kannur International Airport Cochin International Airport Kozhikode International Airport Trivandrum International Airport Bookmark Share question feedback Type: MCQ Single    |    Marks: +1/-0   |    Topic: General Awareness > Static    |   Time Spent:  0 Mins 11 Secs 24.  The Supreme Court can issue writs under which Article(s) of the constitution? A.  Article 32 B.  Article 139 C.  Article 226 D.  Both 1 and 2 Status :   Correct

Answer : D View Solution The Supreme Court can issue writs under Articles 32 and 139 of the constitution. The type of writs include Habeas Corpus, Mandamus, Prohibition, Certiorari and Quo warranto.

Article 32

Enforcements of Fundamental Rights

Article 139

Enforcements of rights excluding Fundamental Rights

Article 226

Gives power to High Courts to issue writs

 NOTE : Article 226 gives power to high courts to issue writs, not to Supreme Court. Bookmark Share question feedback Type: MCQ Single    |    Marks: +1/-0   |    Topic: General Awareness > Static    |   Time Spent:  0 Mins 21 Secs 25.  Who among the following has won the Mohammed Rafi Award 2018? A.  Laxmikant Shantaram Kudalkar B.  Sanjeev Kohli C.  Madan Mohan D.  Yash Raj Status :   Incorrect Given Answer : C Answer : A View Solution    

The 'Mohammed Rafi Award 2018' was given to late music composer Laxmikant Shantaram Kudalkar and playback singer Usha Timothy.  Laxmikant was given the Mohammed Rafi Lifetime Achievement Award consisting of 1 lakh rupees and a trophy. Usha Timothy was honoured with the award and 51000 rupees. The awards were instituted by the NGO Spandan Arts.  Bookmark Share question feedback Type: MCQ Single    |    Marks: +1/-0   |    Topic: General Awareness > Static    |   Time Spent:  0 Mins 15 Secs 26. 

What is the official language of Daman & Diu? A.  Gujarati B.  English C.  Hindi D.  Portrugese  Status :   Correct Answer : A View Solution The officail language of Daman & Diu is Gujarati. Gujarati is the predominant language of Daman & Diu and is the most widely spoken. Bookmark Share question feedback Type: MCQ Single    |    Marks: +1/-0   |    Topic: General Awareness > Static    |   Time Spent:  0 Mins 21 Secs 27.  Who invented the revolver? A.  Edward Jenner B.  Alexander Fleming C.  Albert Einstein D.  Samuel Colt Status :   Correct Answer : D View Solution Samuel Colt was an American Industrialist and businessman who is credited with the invention of Revolver. Bookmark Share question feedback Type: MCQ Single    |    Marks: +1/-0   |    Topic: General Awareness > Static    |   Time Spent:  0 Mins 15 Secs 28.  Which of the following cities is known as "City of Lakes, " the Venice of the East, " or the "Kashmir of Rajasthan?  A.  Jaipur B.  Udaipur

C.  Kota D.  Jodhpur Status :   Correct Answer : B View Solution Udaipur is the city which is called the "City of Lakes, " the Venice of the East, " or the "Kashmir of Rajasthan. This is because of the many lakes are there in the city which add to the beauty of Udaipur. Bookmark Share question feedback Type: MCQ Single    |    Marks: +1/-0   |    Topic: General Awareness > Current Affairs   |    Time Spent:  0 Mins 12 Secs 29.  What is the theme of the National Girl Child Day (NGCD-2019)? A.    Beti Bachao, Beti Padhao  B.  Protect, Educate and Value Girls C.  Protect Future, Protect Girl D.  Empowering Girls for a Brighter Tomorrow Status :   Incorrect Given Answer : A Answer : D View Solution The National Girl Child Day (NGCD) is celebrated every year on January 24 in India to raise awareness against the social stigma and discrimination faced by girl. The day also highlights the need to protect girl child who is prone to sex selective abortions, sexual abuse and trafficking in India. The 2019 theme “Empowering Girls for a Brighter Tomorrow” was celebrated with objectives of generating awareness on the issue of declining Child Sex Ratio (CSR) and create a positive environment around valuing the girl child. Bookmark Share question feedback Type: MCQ Single    |    Marks: +1/-0   |    Topic: General Awareness > Current Affairs   |    Time Spent:  0 Mins 12 Secs 30.  Which two countries officially quit the United Nations Educational, Scientific and Cultural Organization (UNESCO)?

A.  Italy and Libya  B.  Argentina And Algeria C.  Brazil And Cuba D.  United States And Israel Status :   Incorrect Given Answer : C Answer : D View Solution The United States and Israel officially quit the United Nations Educational, Scientific and Cultural Organization (UNESCO) with the close of 2018. The countries had announced their decisions in 2017 of withdrawing from the agency, accusing it of bias against Israel. Bookmark Share question feedback Type: MCQ Single    |    Marks: +1/-0   |    Topic: General Awareness > Current Affairs   |    Time Spent:  0 Mins 22 Secs 31.  Who became the first Hindu woman to have been appointed as a civil judge in Pakistan? A.  Suman Kumari B.  Aashi Devi C.  Sunita Singh D.  Barkha Gupta Status :   Unanswered Answer : A View Solution Suman Kumari has become the first Hindu woman to have been appointed as a civil judge in Pakistan after passing an examination for induction of judicial officers.   Bookmark Share question feedback Type: MCQ Single    |    Marks: +1/-0   |    Topic: General Awareness > Current Affairs   |    Time Spent:  0 Mins 10 Secs 32.  In which city of India IAF carried 'Vayu Shakti' exercise? A.  Pokhran

B.  Pathankot C.  Ghaziabad D.  Gwalior Status :   Correct Answer : A View Solution IAF conducted mega exercise 'Vayu Shakti' in Pokhran, Rajasthan. The exercise was carried out day and night where the IAF showcased firepower capability of indigenously developed platforms and efficacy of missiles. Bookmark Share question feedback Type: MCQ Single    |    Marks: +1/-0   |    Topic: General Awareness > Current Affairs   |    Time Spent:  0 Mins 11 Secs 33.  Which Indian state has topped in budgetary practices, as per latest survey by Transparency International (TI)? A.  Andhra Pradesh B.  Odisha C.  Assam D.  Punjab Status :   Incorrect Given Answer : B Answer : C View Solution As per latest survey conducted by Transparency International (TI), Assam has occupied the top slot in the ranking of best practices followed by states in Budget formulation and is followed by Andhra Pradesh and Odisha. The states which figured lower in the ranking were Meghalaya, Manipur and Punjab. Bookmark Share question feedback Type: MCQ Single    |    Marks: +1/-0   |    Topic: General Awareness > Current Affairs   |    Time Spent:  0 Mins 14 Secs 34.  Which of the following states has recently declared Indus river dolphin as the state's aquatic animal? A.  Punjab

B.  Jammu and Kashmir C.  Haryana D.  Himachal Pradesh Status :   Incorrect Given Answer : D Answer : A View Solution Punjab has recently declared the endangered Indus river dolphin as the state's aquatic animal. The decision was recently taken at a meeting of the state board for wildlife chaired by chief minister Captain Amarinder Singh. Indus Dolphin is an endangered freshwater aquatic mammal found only in Indian and Pakistan in the Beas river. Bookmark Share question feedback Type: MCQ Single    |    Marks: +1/-0   |    Topic: General Awareness > Current Affairs   |    Time Spent:  0 Mins 6 Secs 35.  Which Indian squash player has won the 2019 Seattle Open title? A.  Cyrus Poncha B.  Harinder Pal Sandhu C.  Ramit Tandon D.  Saurav Ghosal Status :   Unanswered Answer : C View Solution Indian squash player Ramit Tandon has won the 2019 Seattle Open title by defeating Mohamed El Sherbini of Egypt in the final in USA. It is his fourth title in his career. The Seattle Open is a Professional Squash Association (PSA) challenger squash Tour event. Bookmark Share question feedback Type: MCQ Single    |    Marks: +1/-0   |    Topic: General Awareness > Current Affairs   |    Time Spent:  0 Mins 12 Secs 36.  Which country's President has been elected as the chairman of the African Union (AU)? A.  Ethiopia B.  South Africa

C.  Egypt D.  Zambia Status :   Incorrect Given Answer : A Answer : C View Solution Egyptian President, Abdel-Fattah el-Sissi has been elected as the chairman of the African Union (AU) at the continental body's summit in Ethiopia. Egyptian President, Abdel-Fattah el-Sissi has been elected as the chairman of the African Union (AU) at the continental body's summit in Ethiopia. Bookmark Share question feedback Type: MCQ Single    |    Marks: +1/-0   |    Topic: General Awareness > Current Affairs   |    Time Spent:  0 Mins 39 Secs 37.  Which international organisation has launched 'The State of the Global Climate' report?  A.  WMO B.  IPCC C.  IUCN D.  UNEP Status :   Correct Answer : A View Solution The State of the Global Climate' report has been recently launched by the World Meteorological Organization (WMO) Bookmark Share question feedback Type: MCQ Single    |    Marks: +1/-0   |    Topic: General Awareness > Current Affairs   |    Time Spent:  0 Mins 22 Secs 38.  ISRO has successfully launched EMISAT and 28 customer satellites by which launch vehicle? A.  PSLV-C44 B.  PSLV-C46 C.  PSLV-C47

D.  PSLV-C45 Status :   Incorrect Given Answer : C Answer : D View Solution On April 1, ISRO has successfully launched India’s latest EMISAT defence satellite from the Sriharikota spaceport. In today’s mission, the rocket PSLV-C45 flew in a new configuration called “QL”. Bookmark Share question feedback Type: MCQ Single    |    Marks: +1/-0   |    Topic: General Awareness > Current Affairs   |    Time Spent:  0 Mins 9 Secs 39.  Who has won the 2019 Formula 1 Bahrain Grand Prix tournament? A.  Valtteri Bottas B.  Sebastian Vettel C.  Charles Leclerc D.  Lewis Hamilton Status :   Correct Answer : D View Solution Lewis Hamilton, a British racing driver, has won the 2019 Formula 1 Bahrain Grand Prix tournament after an engine problem hit runaway leader Charles Leclerc's Ferrari late in the race. Bookmark Share question feedback Type: MCQ Single    |    Marks: +1/-0   |    Topic: General Awareness > Current Affairs   |    Time Spent:  0 Mins 16 Secs 40.  India has recently evacuated peacekeeping CRPF contingent from which of the following African countries? A.  Tunisia B.  Libya C.  Kenya D.  Ethiopia

Status :   Incorrect Given Answer : C Answer : B View Solution India has recently evacuated its entire contingent of peacekeeping forces comprising 15 CRPF personnel from Tripoli after the situation suddenly worsened in Libya. Bookmark Share question feedback

Related Documents


More Documents from "Raia Redila-San Miguel"